I'm very confused with this specific question, one of the few LR that I have ever blind reviewed and been unable to get correct. It's a weaken question but the correct answer ...
http://7sage.com/lsat_explanations/lsat-28-section-3-question-12/
I really do not understand why the answer is C, and the question stem also seems to be very confusing. Are we suppose to find one option that is "must be false", or there are four " ...
hello everyone, this is a question that was asked but there was no clear answer. The answer choice for A, if you take the contraceptive it matches the flaw. Is A only ...
Got this one right by POE but had a tough time being OK with A. It is definitely something I considered as a flaw in the reasoning. However, the use of "the identity of the practical joker" in the stimulus (which we accept as true and cannot contradict) ...
Hey, guys. I think I got this one but I'm seeking clarification. I was between A, B and C and chose A. I think C is correct because a "property owner" is defined in lines 15-21 and A and B are incorrect because they describe how one can obtain ownership. A ...
My big issue with this question is about why B is the correct answer. It seems to equate "exploiting" with "destroy" and I'm not sure how reasonable of an assumption that is to make. Since this is a logically inferred question, I assumed that the right ...
This question tripped me up a bit, not because I didn't understand what I was being asked to do, but because I couldn't really differentiate between some of the answer choices.
Hi everyone, question on how to approach this question. The question gives two arguments, Jane's and Maurice's. The question stem states, "Which one of the following, if true, most strengthens Jane's argument?" So after reading this question stem we know ...
Can someone help me out with identifying the flaw here? There's not an explanation video, so I've linked the question bank. Preptest C is at the bottom.
I don't really understand answer choice A) if there is no overwhelming evidence for or against a hypo. then one should "suspend judgment as to its truth". What does it mean by "as to its truth"? Anyone could help?
So I can see why C is definitely a better answer choice than all the rest. However, I find myself confused by the fact that I am not sure how we can tell that the author thinks that doctrine of precedent is a "useful tool" here. Can ...
Okay, so I'm currently running through SA's and PSA's and ran into this and actually gave it some time to sit with me (after looking it up on the actual explanation video, powerscore forums, manhattan prep forums) and I've come to rationalizing D as such:< ...
Hi everyone. I had difficulty seeing which sentence was the conclusion in this question (the Guam/spider one). I know the first sentence is context. Thank you!
Is this a correlation-causation argument because it assumes that the increase in high school dropouts is the only thing that is causing the increase in recruitment among 18 year olds? And why would the author draw such a conclusion?
Sharing afew LR problems with conditional language I'd saved over the past few months:
PT09 S2 Q13
PT51 S1 Q21
PT51 S3Q14
PT51 S3 Q19
PT60 S3 Q12
PT71 S3 Q11
PT71 S3 Q17